helphelphelphelphelphelphelphelphelphelphelphelphelphelphelp

Helphelphelphelphelphelphelphelphelphelphelphelphelphelphelp

Answers

Answer 1
It would be the first one
Answer 2

Answer:

Its mass or size.

The amount possessed by an object directly upon the square of the speed. It increases the kinetic energy.


Related Questions

Which list of numbers could be used to name each point plotted on the number line?

A nuimber line goes from 3.45 to 3.6 in increments of 0.01. Blanks are located as follows: above the first tick mark after 3.45, above the second tick mark after 3.45, above the third tick mark after 3.5, and above the first tick mark after 3.55.

Answers

Answer:

Step-by-step explanation:

numbers are {3.46,3.47,3.48,3.49,3.50,3.51,3.52,3.53,3.54,3.55,3.56,3.57,3.58,3.59}

label the angle that is 57° in this 2-dimensional figure.

Answers

The measure of the angle that is 57° in this 2-dimensional figure is the angle vertically opposite to the angle 57  (the angle formed at X)

Line and angles

From the given diagram, we have the following:

The two angles on the straight line are 123 degrees and 57 degrees.

<F + 57 = 180

<F = 180 - 57

<F = 123 degrees

Hence the measure of the angle that is 57° in this 2-dimensional figure is the angle vertically opposite to the angle 57  (the angle formed at X)

Learn more on lines and angles here: https://brainly.com/question/25770607


A candy maker produces 465 pounds of chocolate bites. It is placed in equal amounts into 145 empty bags. Each bag can hold up to 3 pounds. How many pounds of chocolate bites are left over after the bags are filled?​

Answers

Answer:

30 pounds are left over after the bag filled

Step-by-step explanation:

145×3

= 435

now,

465-435

= 30

Find the value of x. Round your answer to the nearest tenth.

Answers

16.4 that is the answer! Use tan(64)*7

PLEASE HELP ME
( I WILL MARK BRAINLIEST )

Answers

the numbers are 15, 17 and 19

I don’t really know how to do this , can someone help?

Answers

Answer:

A 67 inches is the correct answer

[tex]\text{Given that,}\\\\\text{Area of the rectangle,}~ A = 344 ~ in^2\\\\\text{Width of the rectangle,}~w = 16 ~ in\\\\\text{Length of the rectangle,}~ l=?\\\\~~~~~~A = l \times w \\\\\implies l= \dfrac{A}w =\dfrac{344}{16}=21.5 ~in\\\\\text{Perimeter of the rectangle} = 2(l+w) = 2(21.5+16)=75~ in[/tex]

7.4 Practice
A medicine is effective on 70% of patients.
The table shows 30 randomly generated
numbers from to 999. Use the table to
estimate the probability of the event.
1. The modicine is effective on at least two
of the next three patients.
028837 618 205 984
724 301249 946 925
042 | 113 | 696 985 632
312085 | 997 198 | 398
117 240 853:373 597
606 077 016 012
695
2. The medicine is effective on none
of the next three patients.
Design and use a simulation to find the experimental probability.
3. A bowler hats the headpin 90% of the time that all ten pins are standing.
What is the experimental probability that the bowler hits the headpin
exactly four of the next five times that all ten pins are standing?

Answers

The simulation of the medicine and the bowler hat are illustrations of probability

The probability that the medicine is effective on at least two is 0.767The probability that the medicine is effective on none is 0The probability that the bowler hits a headpin 4 out of 5 times is 0.3281

The probability that the medicine is effective on at least two

From the question,

Numbers 1 to 7 represents the medicine being effective0, 8 and 9 represents the medicine not being effective

From the simulation, 23 of the 30 randomly generated numbers show that the medicine is effective on at least two

So, the probability is:

p = 23/30

p = 0.767

Hence, the probability that the medicine is effective on at least two is 0.767

The probability that the medicine is effective on none

From the simulation, 0 of the 30 randomly generated numbers show that the medicine is effective on none

So, the probability is:

p = 0/30

p = 0

Hence, the probability that the medicine is effective on none is 0

The probability a bowler hits a headpin

The probability of hitting a headpin is:

p = 90%

The probability a bowler hits a headpin 4 out of 5 times is:

P(x) = nCx * p^x * (1 - p)^(n - x)

So, we have:

P(4) = 5C4 * (90%)^4 * (1 - 90%)^1

P(4) = 0.3281

Hence, the probability that the bowler hits a headpin 4 out of 5 times is 0.3281

Read more about probabilities at:

https://brainly.com/question/25870256

Alyssa and Gabriel play the following game. Alyssa rolls a die. If she rolls a​ 1, 2, or​ 3, Gabriel gives Alyssa ​$4. If Alyssa rolls a 4 or​ 5, Gabriel gives Alyssa ​$. ​However, if Alyssa rolls a​ 6, she gives Gabriel ​$23.
​a) Determine​ Alyssa's expectation.
​b) Determine​ Gabriel's expectation.

Answers

Determine Alyssa’s expectations

Which is the net of this solid?
O
Which is the net of this solid?

Answers

Answer:

I think the A is the best answer

Step-by-step explanation:

PLEASE MARK ME BRAINLIEST IF MY ANSWER IS CORRECT PLEASE

PLEASE please please please please please please I need it Badly

What is the slope of the line that passes
through (7,3) and (-2,4)?
O 7
02
- 9
0-
OS

Answers

[tex]\qquad\qquad\huge\underline{{\sf Answer}}♨[/tex]

Let's calculate the slope ~

[tex]\qquad \sf  \dashrightarrow \: \dfrac{y_2 - y_1}{x_2 - x_1} [/tex]

[tex]\qquad \sf  \dashrightarrow \: \dfrac{3 - 4}{7 - ( - 2)} [/tex]

[tex]\qquad \sf  \dashrightarrow \: \dfrac{ - 1}{9} [/tex]

That's the required slope, I hope it helps~

HELP

explain what the answer is

Answers

Answer:10/7 and mixed number form is 1 3/7

A rectangle has a length of (x + 4) feet and a width of (x + 7) feet. The expression
2(x+)+2(x + 7) represents the perimeter of the rectangle.
Which expression also represents the perimeter of the rectangle, in feet?
A (x + 4) + (x + 4) + (x + 7) + (x + 7) W
B (x + 4)(x + 4)(x + 7)(x + 7)
C 4x + 11
D 22x

Answers

Answer:

A

Step-by-step explanation:

No need. It’s simple.

B can’t be it because it uses multiplication.

C can’t be it because it makes no sense.

D also makes no sense.

how many roots does y=x2-6x+9 have

Answers

Answer:

One root ; x = 3

Step-by-step explanation:

( a ± b )² = a² ± 2ab + b²

~~~~~~~~~

y = x² - 6x + 9

x² - 6x + 9 = x² - 2(x)(3) + 3² = ( x - 3 )²

( x - 3 )² = 0 ⇒ x - 3 = 0 ⇒ x = 3

What is the slope of the line given by the equation below?
y-9 = 15(x - 5)
O A. -5
O B. 9
O C. -9
O D. 15

Answers

Answer:

D

Step-by-step explanation:

this would simplify, in slope-intercept form, as y= 15x - 66

5
0 = - in
=
in quadrant II
Given cos =
Find sin
3

Answers

Answer:

sinΘ = [tex]\frac{2}{3}[/tex]

Step-by-step explanation:

using the identity

sin²x + cos²x = 1  ( subtract cos²x from both sides )

sin²x = 1 - cos²x ( take square root of both sides )

sinx = ± [tex]\sqrt{1-cos^2x}[/tex]

given

cosΘ = - [tex]\frac{\sqrt{5} }{3}[/tex] , then

sinΘ = ± [tex]\sqrt{1-(-\frac{\sqrt{5} }{3})^2 }[/tex]

        = ± [tex]\sqrt{1-\frac{5}{9} }[/tex]

        = ± [tex]\sqrt{\frac{4}{9} }[/tex]

        = ± [tex]\frac{2}{3}[/tex]

since Θ is in quadrant II where sinΘ > 0 , then

sinΘ = [tex]\frac{2}{3}[/tex]

Find the slope of the line that passes through the two given points:

(-1, 4) and (5, 3)
A line starting in the third quadrant, crossed the x axis twice, and then ending in the first quadrant

Answers

Answer:

[tex] \frac{ - 1}{6} [/tex]

Step-by-step explanation:

[tex]slope \: = \frac{y2 - y2}{x2 - x1} \\ slope = \frac{3 - 4}{5 - - 1} \\ sope = \frac{ - 1}{ \: \: \: 6} [/tex]

What is the median of this data set?
{29, 22, 23, 31, 29, 29, 22, 23}

Answers

Answer:

The median is 26

Step-by-step explanation:

A "median" is equivalent to when the terms are arranged in the proper order, whichever term is in the "middle".

So- first, arrange the terms in ascending order.

[22, 22, 23, 23, 29, 29, 29, 31]

There are an even number- eight- terms in total, so we will take the average of the two "middle" terms.

(29+23)/2; 29 and 23 are the "middle" terms, and there are two of them.

(52)/2 = 26

Therefore, the median is 26.

I hope this helped! :)

Hello!

In order to find the median of a dataset, we should first order the numbers in the dataset from least to greatest:

29, 22, 23, 31, 29, 29, 22, 23

The smallest number is 22, then 23, then 29, then 31:

22, 22, 23, 23, 29, 29, 29, 31

How many numbers are in this set? 8. An even number!

So we find the mean of the 2 numbers in the middle (23 & 29)

We add these numbers and then divide by 2 (because we have 2 numbers)

[tex]\bf{\displaystyle\frac{23+29}{2}}[/tex]

[tex]\bf{\displaystyle\frac{52}{2}}[/tex]

Which is equal to

[tex]\boxed{\bigstar{\text{Answer:{\boxed{\boxed{26}}}}}}[/tex]

Hope everything is clear.

Let me know if you have any questions!

#KeepLearning

:-)

When using synthetic division you must switch the signs in the divisor

Correct answer:

True

False


















b a dddddddies hmuuuuuuuuuuuu XD

Answers

True, you switch the sign when you write the divisor so that you can add

Based on the following circle, solve for the value of a.

Answers

The answer is 10
I did the math out
(8a - 9) = (7a + 1)

If radius =5 units and x = 12 units what is the volume of the cylinder

Answers

[tex]\qquad\qquad\huge\underline{{\sf Answer}}♨[/tex]

It's given that,

radius (r) = 5 units

height (x) = 12 units

Now, according to formula ~

[tex]\qquad \sf  \dashrightarrow \: v = \pi {r}^{2}x[/tex]

[tex]\qquad \sf  \dashrightarrow \: v = \frac{22}{7} \times {5}^{2} \times 12[/tex]

[tex]\qquad \sf  \dashrightarrow \: v = \frac{22}{7} \times 25 \times 12[/tex]

[tex]\qquad \sf  \dashrightarrow \: v = \frac{6600}{7} [/tex]

[tex]\qquad \sf  \dashrightarrow \:v \approx 942.86 \: \: unit {}^{3} [/tex]

In a math competition, every question that is answered correctly gets a score of 4. Wrong gets a score of -2. and not answered gets a score of -1. Ali got a score of 106 with 4 questions answered incorrectly and 6 questions not being answered. Many questions were answered correctly by........
please explain it​

Answers

Answer:

Step-by-step explanation:

First we need to find the score of the incorrect answers and the questions that were not answered.

Score for the incorrect answers = 4*(-2) = -8

Score for the questions not answered = 6*(-1) = -6

Score for the questions answered correctly = 106 - [ (-8) + (-6) ]

                                                                       = 106 -[-14]

                                                                        = 106 + 14

                                                                        = 120

Number of questions answered correctly = total score for questions that were answered correctly ÷ score of 1 correct answer

= 120 ÷ 4

= 30

30 questions were answered correctly.

SOLVE THE INEQUALITY FOR X - 27 < 12

Answers

Answer:

x < 39

Step-by-step explanation:

x - 27 < 12

Add 27 to both sides.

x - 27 + 27 < 12 + 27

x < 39

[tex]\qquad\qquad\huge\underline{{\sf Answer☂}}[/tex]

Let's solve the inequality ~

[tex]\qquad \sf  \dashrightarrow \: x - 27 < 12[/tex]

[tex]\qquad \sf  \dashrightarrow \: x < 12 + 27[/tex]

[tex]\qquad \sf  \dashrightarrow \: x < 39[/tex]

Help help help ASAP

Answers

Answer:

-8

Step-by-step explanation:

x+2=2(x-1)

_

3

3x+6=2x-2

3x-2x=-2-6

x=-8

What is the slope-intercept equation for this line?

y = [?]x + ( )

Answers

Answer:

y=-x-3

Step-by-step explanation:

The fence around Jan's backyard
is 45 feet long and 30 feet wide.
What is the perimeter of the fence?
What is the area of the yard?

Answers

Answer:

Perimeter : 150

Area : 1,350

1. Suppose you pay back $680 on a $650 loan you had for 75 days. What was your simple annual interest rate?

Answers

well, let's assume a year has 365 days, so 75 days is simply 75/365 of a year.  We also know that your loan was $650 and you paid back $680 or namely 30 bucks extra, so the interest on those $650 is really 30 bucks.

[tex]~~~~~~ \textit{Simple Interest Earned} \\\\ I = Prt\qquad \begin{cases} I=\textit{interest earned}\dotfill &\$30\\ P=\textit{original amount deposited}\dotfill & \$650\\ r=rate\to r\%\to \frac{r}{100}\\ t=years\to \frac{75}{365}\dotfill &\frac{15}{73} \end{cases} \\\\\\ 30 = (650)(\frac{r}{100})(\frac{15}{73})\implies 30=\cfrac{9750r}{7300}\implies 30=\cfrac{195r}{146}\implies 4380=195r \\\\\\ \cfrac{4380}{195}=r\implies \stackrel{\%}{22.46}\approx r[/tex]

4 2/3 divided by 1 1/9

Answers

Answer:

4.2 or 21/5

Step-by-step explanation:

i used demos

Answer:

4 1/5

[tex]\frac{14 * 9}{3 * 10} = \frac{126}{30}~which~is~also~4\frac{1}{5}[/tex]

How many distinct 3 digit code can i create such that this code is divisible by 4.
For example these codes are rejected since they have repeating numbers/less than 3 digits:
024/100/112/996/444

Answers

The other answer is just wrong.

There are 9•9•8 = 648 distinct 3-digit codes. The first digit can be any numeral from 1-9, the next digit can be any from 0-9 minus the one used in the first position, and the last digit can be any from 0-9 minus both the numerals used in the first two positions.

But that doesn't even account for the divisibility constraint.

Let the code be [tex]abc[/tex]. We can expand this as

[tex]100a + 10b + c[/tex]

In order for this to be divisible by 4, we observe that

[tex]100a + 8b + 2b + c = 4 (25a + 2b) + (2b+c)[/tex]

so we only need [tex]2b+c[/tex] to be divisible by 4.

The last digit must be even, so there are only 5 choices for the last digit. I list the possibilities and outcomes below. For some integer [tex]k[/tex], we need

[tex]c=0 \implies 2b=4k \implies b=2k[/tex]

[tex]c=2 \implies 2b+2=4k \implies b = 2k-1[/tex]

[tex]c=4 \implies 2b+4 = 4k \implies b = 2(k-1)[/tex]

[tex]c=6 \implies 2b+6 = 4k \implies b = 2k-3[/tex]

[tex]c=8 \implies 2b+8=4k \implies b = 2(k-2)[/tex]

Ignoring [tex]a[/tex] for the moment, in the cases of [tex]c\in\{0,4,8\}[/tex], [tex]b[/tex] is also even. This leaves 3 choices for [tex]c[/tex] and 2 choices for [tex]b[/tex].

Likewise, in the cases of [tex]c\in\{2,6\}[/tex], [tex]b[/tex] is odd. This leaves 2 choices for [tex]c[/tex] and 5 choices for [tex]b[/tex].

Now taking into account the choice for [tex]a[/tex], we have the following decision tree.

• If [tex]a\in\{2,6\}[/tex] and [tex]c\in\{0,4,8\}[/tex], then [tex]b\in\{0,2,4,6,8\}\setminus\{a,c\}[/tex] - a total of 2•3•3 = 18 codes.

• If [tex]a\in\{4,8\}[/tex] and [tex]c\in\{0,4,8\}\setminus\{a\}[/tex], then [tex]b\in\{0,2,4,6,8\}\setminus\{a,c\}[/tex] - a total of 2•2•3 = 12 codes.

• If [tex]a\in\{2,6\}[/tex] and [tex]c\in\{2,6\}\setminus\{a\}[/tex], then [tex]b\in\{1,3,5,7,9\}\setminus\{a,c\}[/tex] - a total of 2•1•5 = 10 codes.

• If [tex]a\in\{4,8\}[/tex] and [tex]c \in\{2,6\}[/tex], then [tex]b\in\{1,3,5,7,9\}[/tex] - a total of 2•2•5 = 20 codes.

• If [tex]a\in\{1,3,5,7,9\}[/tex] and [tex]c\in\{0,4,8\}[/tex], then [tex]b\in\{0,2,4,6,8\}\setminus\{c\}[/tex] - a total of 5•3•4 = 60 codes.

• If [tex]a\in\{1,3,5,7,9\}[/tex] and [tex]c\in\{2,6\}[/tex], then [tex]b\in\{1,3,5,7,9\}\setminus\{a\}[/tex] - a total of 5•2•4 = 40 codes.

Hence there are a total of 18 + 12 + 10 + 20 + 60 + 40 = 160 codes.

7. Frank wants to paint his room in the school colors of maroon and white. The floor and
ceiling will be white, and all the walls will be maroon. The door will also be white. If one
gallon of paint covers 400 sq ft, how many gallons of each color will he need?
A. 1 gallon white, 1 gallon maroon
B. 1gallon white, 2 gallons maroon
C. 2 gallons white, 2 gallons maroon
D. 2 gallons white, 3 gallons maroon
10 ft
3 ft
7 ftlo
12 ft
12 ft

Answers

Frank would need two gallons of maroon painting and one gallon of white painting to paint all his room.

How much white painting is required?

Area of the ceiling:

12 feet x 12 feet = 144 squared feet

Assuming the floor has the same area, the total area for the floor and ceiling is 288 square feet.

Area of the door:

3 feet x 7 feet = 21 feet

Total area in white 144 feet + 21 feet = 165 square feet

Total of gallons

1 gallon equals 400 square feet, which means only one gallon of white painting is required.

How much maroon painting is required?

Area of one wall:

12 feet x 10 feet = 120 feet120 feet x 4 walls = 480 square feet480 square feet - 21 feet (door) = 459 square feet

Considering 1 gallon equals 400 square feet, a total of 2 gallons would be required.

Note: This question is incomplete because the image of the room is missing; below I attach the missing section.

Learn more about area in: https://brainly.com/question/16151549

Easy math 2 pictures

Answers

Answer:

Problem 3: 51°

Problem 5: 5 feet

Step-by-step explanation:

Image 1:

We are asked to find the missing angle. First off, we know that the full semi-circle equals 180 degrees. knowing this, subtract the second angle from 180 to get the smaller angle.

180 - 129 = 51

The unkown angle is 51 degrees.

Image 2:

We are asked to find the missing side. Apply the numbers of the known sides to the Pythagorean Theorem.

a² + b² = c²

a² + 12² = 13²

a² + 144 = 169

a² = 25

a = 5

The missing side is 5 feet.

Other Questions
What is the slope of this line whose equation is y=1/6x-1/2 determine the common factors and HCF for each of the following terms1) w, 5w2)5bc ,2c, 3cd3)4a b, 8b c ,6bcdPLS HELP What is the effectiveness and time to take effect of Claritin? -5X^2 + 3X = -3X^2 + 3X - 50 solve for x The rabies virus is capable of entering the brain with the blood of a victim of the bite of an infected animal. Propose a mechanism for this to happen. Hal is going over the credit scores he received from the three major credit bureaus. He Experian score is 711, his Equifax score is 736, and his TransUnion score is 736. What is the mode of Hals credit scores? (Round to the nearest whole point, if applicable. ) a. 736 b. 728 c. 723 d. There is no mode in this group. Examine the following piecewise function.A graph formed of straight line segments between the following points: (negative 8, 3), (negative 2, 8), (2, 8), (3, 0), and (7, negative 7). 2018 StrongMind. Created using GeoGebra.Which statements are true?Select all that apply.1.The function is increasing over the interval 2x32.The function is increasing over the interval 2x3.3.The function is increasing over the interval 8x2.4.The function is increasing over the interval 8x2. Make 2 full sentences using the scientific term "homoozygous" and 2 full sentences for "heterozygous (Side note: i had to add an extra O to the first thing so its not considered a offensive word-Thank you for your help!)im getting really annoyed i have posted this 2 times and bots keep answering and ruining it lol Dr. Bayer measured the masses of newborn twins. Baby A weighs 2.8 kilograms. Baby B weighs 2,950 grams. What is the difference between the masses of these newborn twins in grams? OPTIONS The fact that large numbers of south asians migrated to great britain after the end of british rule could indirectly be used as evidence to support the authors arguments mostly because the migrations of former colonial subjects to imperial metropoles in the late twentieth century demonstrate that:. A scientist wishes to obtain data about a star more than 15 light-years from Earth. Which toolwould probably provide the most useful data?a. A space probeb. A space observatoryc. A satellite orbiting the moond. A spacecraft with a human crew IdentificarIdentify the place where these activities would take place.un caf,el centro,un cine,un gimnasio,una montaa,un museo,una piscina,una plaza,un restaurante,1. Tomamos una limonada.2.Vemos una pelcula. 3.Practicamos deportes.4.Hay muchos monumentos.5.Comemos tacos y fajitas.6. Nadamos y tomamos el sol.7.Hay mucho trfico.8. Miramos pinturas (paintings) de Diego Rivera y Frida Kahlo. Find someone, your relative, friend, or acquaintance who has been to a second hand shop. Ask him/her about the experience of visiting the second hand shop and prepare a report in about one hundred words If QSRQST and QT=65, what is QR? solve equation find x 6x+6=10x-2 What can you discern about the nature of work on miguel street based on your close reading of the first chapter What part of the criminal justice system is responsible for investigating crimes?correctionsthe courtsparole agencieslaw enforcement Cooper is throwing a dinner party. He says about 67 people are coming to the party. He is paying a company to make the food, and they said each person would cost about $15. To add drinks, each person would cost $3. How much money will Cooper be spending for food and drinks at his party if you round the number of people to the nearest tenth digit? What is a way you can solve this estimation problem? Select all that apply. Round the $15 to $20, the $3 to $5 and the 67 to 70 before you do anything else. Round the 67 to 70, add the $15 and the $3 and then multiply the 2 answers. Round 67 to 70. Multiply 70 by 15 and 70 by 3 and then add the answers. Add the amount for food and drinks then multiply by how many people there are. Water is a ______ molecule and the bond between the h-o is a __ ______ bond.i know the last 2 is polar covalent bond but i don't know the first blank. What is a mental disorder characterized by overeating without vomiting?O Anorexia nervosa Binge eating disorderO Bulimia nervosaO Anxiety eating disorder